5
$\begingroup$

Let $x,y$ be generators for the free group $F_2$. It's known that $Aut(F_2)$, and hence $Out(F_2)$ preserves the conjugacy class of the subgroup $\langle[x,y]\rangle$ generated by $[x,y]$ (This conjugacy class is in some contexts called the nielsen invariant)

On the other hand, if we view $x,y$ as topological generators of $\hat{F_2}$ (hence fixing an embedding $F_2\hookrightarrow\hat{F_2}$), then $Out(\hat{F_2})$ does not have the same property of preserving the conjugacy class of $\langle [x,y]\rangle$ (this follows from the fact that $\hat{F_2}$ has the strong lifting property). By the strong lifting property I mean that for any finite group $G$ and two surjections $\varphi,\psi : \hat{F_2}\rightarrow G$, there is an $\alpha\in Aut(\hat{F_2})$ such that $\psi = \varphi\circ\alpha$. If $Aut(\hat{F_2})$ were to preserve the Nielsen invariant, then the image of the conjugacy class of $\langle [x,y]\rangle$ in $G$ would be the same for any surjection $\hat{F_2}\rightarrow G$, which is to say that all commutators of generating pairs of finite groups generate conjugate subgroups. This is easily verified to be false - the smallest example is the alternating group $A_5$ - it has two generating pair with commutators of order 3 and 5 respectively.

My question is - What is the stabilizer of the conjugacy class of $\langle[x,y]\rangle$ in $Out(\hat{F_2})$?

The stabilizer should be a closed subgroup, lets call it $S$ - could it be $\overline{GL_2(\mathbb{Z})}\cong\overline{Out(F_2)}\cong \widehat{Out(F_2)}\subset Out(\hat{F_2})$?

If not, can we describe the difference $S/\overline{GL_2(\mathbb{Z})}$ somehow?

$\endgroup$
3
  • $\begingroup$ Could you say a little more about the strong lifting property? $\endgroup$
    – HJRW
    Sep 5, 2015 at 20:43
  • $\begingroup$ @HJRW I've explained it a bit more as an edit. $\endgroup$
    – Will Chen
    Sep 8, 2015 at 16:11
  • $\begingroup$ @HJRW Also I should have said $\hat{F_2}$ has the strong lifting property (not $Aut(\hat{F_2})$). I've edited to correct that. $\endgroup$
    – Will Chen
    Sep 8, 2015 at 16:38

0

Your Answer

By clicking “Post Your Answer”, you agree to our terms of service and acknowledge you have read our privacy policy.

Browse other questions tagged or ask your own question.